You are on page 1of 26

POLITICAL LAW REVIEW

Atty. Jacinto Jimenez

ARTICLE VII
EXECUTIVE DEPARTMENT

SECTION 1
PHILCONSA VS. ENRIQUEZ
EXECUTIVE.

General Appropriations Act of 1994 RA 7663 was passed and approved by Congress. It authorized the
members of Congress to propose and identify projects in Pork Barrels, allotted to them and to
realign their respective operating budgets. Also, the president vetoed some provisions and imposed
conditions on an item in the appropriations bill
Petitioners assail the constitutionality of the CDF. Petitioners claim that the power given to the
members of Congress to propose and identify the projects and activities to be funded by the
Countrywide Development Fund is an encroachment by the legislature on executive power, since said
power in an appropriation act is in implementation of a law. They argue that the proposal and
identification of the projects do not involve the making of laws or the repeal and amendment thereof,
the only function given to the Congress by the Constitution

ISSUE: Do they have legal standing?


SC: YES.
Where a veto is claimed to have been made without or in excess of the authority vested on the President,
the issue of an impermissible intrusion of the Executive into the domain of the Legislature arises. To the
extent the powers of Congress are impaired, so is the power of each member thereof, since his office
confers a right to participate in the exercise of the powers of that institution.
An act of the Executive which injures the institution of Congress causes a DERIVATIVE SUIT, but
nonetheless substantial injury, which can be questioned by any member of Congress. In such case, any
member of Congress can resort to the courts.
It is true that the Constitution provides a mechanism for overriding a veto. Said remedy however is
available only when the presidential veto is based on policy or political considerations. It is NOT
AVAILABLE when the veto is claimed to be ULTRA VIRES. When it is ultra vires, it becomes the duty of the
Court to draw the dividing line where the exercise of executive power ends and the bounds of legislative
jurisdiction begin.
ISSUE: Is the CDF valid?
SC: Under the Constitution, the spending power called by James Madison as "the power of the purse,"
belongs to Congress, subject only to the veto power of the President. The President may propose the
budget, but still the final say on the matter of appropriations is lodged in the Congress.
The power of appropriation carries with it the power to specify the project or activity to be funded under
the appropriation law. It can be as detailed and as broad as Congress wants it to be.
The Countrywide Development Fund is explicit that it shall be used "for infrastructure, purchase of
ambulances and computers and other priority projects and activities and credit facilities to qualified
beneficiaries. . . ." It was Congress itself that determined the purposes for the appropriation.
Executive function under the Countrywide Development Fund involves implementation of the priority
projects specified in the law.
The authority given to the members of Congress is only to propose and identify projects to be
implemented by the President. Under Article XLI of the GAA of 1994, the President must perforce examine
whether the proposals submitted by the members of Congress fall within the specific items of
expenditures for which the Fund was set up, and if qualified, he next determines whether they are in line
with other projects planned for the locality. Thereafter, if the proposed projects qualify for funding under
the Fund, it is the President who shall implement them.
In short, the proposals and identifications made by the members of Congress are merely
recommendatory.

Updated Champ Digests

Fel. Jill. Lauren. Mon. Tara. Charms

POLITICAL LAW REVIEW


Atty. Jacinto Jimenez
The Constitution is a framework of a workable government and its interpretation must take into account
the complexities, realities and politics attendant to the operation of the political branches of government.
Prior to the GAA of 1991, there was an uneven allocation of appropriations for the constituents of the
members of Congress, with the members close to the Congressional leadership or who hold cards for
"horse-trading," getting more than their less favored colleagues. The members of Congress also had to
reckon with an unsympathetic President, who could exercise his veto power to cancel from the
appropriation bill a pet project of a Representative or Senator.
The Countrywide Development Fund attempts to make equal the unequal. It is also a recognition that
individual members of Congress, far more than the President and their congressional colleagues are likely
to be knowledgeable about the needs of their respective constituents and the priority to be given each
project.

WEBB VS. DE LEON


EXECUTIVE POWER.

This is the Hubert Webb- Vizconde Massacre case. Hubert is co-accused in the crime of Rape with
Homicide.
Jessica Alfaro is the star witness.
Alfaro qualified under the Witness Protection Program, RA 6981. Thus she was not included in the
Complaint or Information
Hubert contends that the DOJ failed to include Alfaro in the information for her alleged conspiratorial
participation in the said crime.
Hubert challenges the law, RA 6981 claiming that it constitutes an intrusion into the judicial
prerogative for it is only the court which has the power to discharge an accused as a state witness,
(under the Rules on CrimPro).

ISSUE: Can the DOJ (the executive branch) approve the discharge of a state witness
SC: VALID.
The PROSECUTION OF CRIMES pertains to the executive department, whose principal power and
responsibility is to see to it that our laws are faithfully executed. A necessary component of this power to
execute our laws is THE RIGHT TO PROSECUTE THEIR VIOLATORS. The right to prosecute vests the
prosecutor with a WIDE RANGE OF DISCRETION, and the discretion of whether, what and whom to charge,
the exercise of which DEPENDS ON A SMORGASBOARD OF FACTORS WHICH ARE BEST APPRECIATED BY
PROSECUTORS.
The law is valid in vesting the DOJ with the power to determine who can qualify as a witness and who
shall be granted immunity from prosecution.
Further, the Rules on CrimPro apply only because the courts have already acquired jurisdiction over the
crime and the accused. The discharge of an accused to be a state witness is NOT AN INHERENT JUDICIAL
PREROGATIVE. Courts do so only as part of the exercise of their jurisdiction.
Further, the Rules of CrimPro are not beyond change by legislation designed to improve the administration
of our justice system. For a more effective administration of criminal justice, there was a necessity to pass
the law protecting a witness and granting her certain rights and benefits to ensure their appearance in
court.
Thus, Huberts challenge to the validity of RA 6981 cannot succeed.

Updated Champ Digests

Fel. Jill. Lauren. Mon. Tara. Charms

POLITICAL LAW REVIEW


Atty. Jacinto Jimenez

SECTION 8
Estrada vs Desierto
-

This case concerns the office of the President. Erap alleged that he is the President on leave,
while GMA claims she is the President.
In the May 1998 elections, Erap was elected President, GMA was Vice-President. Erap was then
accused of receiving millions in jueteng money. An impeachment trial was started. The senatorjudges ruled against opening the second envelope which allegedly contained evidence that Erap
had a secret bank account under the name Jose Velarde. The prosecutors walked out and the
people held a rally on EDSA. Chief Justice Davide administered the oath to GMA as President, and
Erap left Malacanang and issued a press statement.1 On the same day, he also signed another
letter.2

Issue: Does the case at bar involve a political question?


Held: NO. The Court has jurisdction to determine whether or not there has been GADLEJ. Respondents
rely on the case of Lawyers League for a Better Philippines and/or Oliver A. Lozano v. President Corazon C.
Aquino, et al. and related cases to support their thesis that since the cases at bar involve the legitimacy of
the government of respondent Arroyo, ergo, they present a political question. These cases are inapplicable
because in those cases, the government was a revolutionary one and EDSA I. In this case, GMAs is not
revolutionary in character. The oath she took at the EDSA shrine is the oath under the 1987 Constitution.
EDSA I involved the exercise of the people power of revolution which overthrew the whole government,
while EDSA II is an exercise of people power freedom of speech and freedom of assembly to petition the
government for redress of grievances. EDSA I presented a political question while EDSA II involves legal
questions and is intra-constitutional.
Issue: Whether Erap is a President on leave while GMA is an Acting President.. or did he resign?
Held: Erap resigned.
For there to be resignation, there must be intent to resign and the intent must be coupled by acts
of relinquishment.
The diary of Exec Secretary Angara serialized in the Inquirer shows the state of mind of Erap. It
reveals that Erap had decided to call for a snap presidential election and stressed he would not be
1

"20 January 2001

STATEMENT FROM
PRESIDENT JOSEPH EJERCITO ESTRADA
At twelve o'clock noon today, Vice President Gloria Macapagal-Arroyo took her oath as President of the
Republic of the Philippines. While along with many other legal minds of our country, I have strong and
serious doubts about the legality and constitutionality of her proclamation as President, I do not wish to be
a factor that will prevent the restoration of unity and order in our civil society.
It is for this reason that I now leave Malacaang Palace, the seat of the presidency of this country, for the
sake of peace and in order to begin the healing process of our nation. I leave the Palace of our people with
gratitude for the opportunities given to me for service to our people. I will not shirk from any future
challenges that may come ahead in the same service of our country.
I call on all my supporters and followers to join me in to promotion of a constructive national spirit of
reconciliation and solidarity.
May the Almighty bless our country and beloved people.
MABUHAY!
(Sgd.) JOSEPH EJERCITO ESTRADA"
2

"Sir:

By virtue of the provisions of Section 11, Article VII of the Constitution, I am hereby transmitting this
declaration that I am unable to exercise the powers and duties of my office. By operation of law and the
Constitution, the Vice-President shall be the Acting President.
(Sgd.) JOSEPH EJERCITO ESTRADA"
Updated Champ Digests

Fel. Jill. Lauren. Mon. Tara. Charms

POLITICAL LAW REVIEW


Atty. Jacinto Jimenez

a candidate. The proposal for a snap election for president in May where he would not be a
candidate is an indicium that petitioner had intended to give up the presidency even at that time.
Erap also expressed no objection to the suggestion for a graceful and dignified exit but said he
would never leave the country. He also said, Pagod na pagod na ako. Ayoko na masyado nang
masakit. Pagod na ako sa red tape, bureaucracy, intriga. (I am very tired. I don't want any more
of this it's too painful. I'm tired of the red tape, the bureaucracy, the intrigue.) Erap and GMA
were also entering into negotiations about Erap resigning but they were cut short when GMA had
already taken the oath before she could sign their agreement.
In sum, we hold that the resignation of the petitioner cannot be doubted. It was confirmed by his
leaving Malacaang. In the press release containing his final statement, (1) he acknowledged the
oath-taking of the respondent as President of the Republic albeit with reservation about its
legality; (2) he emphasized he was leaving the Palace, the seat of the presidency, for the sake of
peace and in order to begin the healing process of our nation. He did not say he was leaving the
Palace due to any kind inability and that he was going to re-assume the presidency as soon as
the disability disappears: (3) he expressed his gratitude to the people for the opportunity to
serve them. Without doubt, he was referring to the past opportunity given him to serve the
people as President (4) he assured that he will not shirk from any future challenge that may
come ahead in the same service of our country. Petitioner's reference is to a future challenge
after occupying the office of the president which he has given up; and (5) he called on his
supporters to join him in the promotion of a constructive national spirit of reconciliation and
solidarity. Certainly, the national spirit of reconciliation and solidarity could not be attained if he
did not give up the presidency. The press release was petitioner's valedictory, his final act of
farewell. His presidency is now in the part tense.
Is Erap only temporarily unable to act as President? This involves an examination of Article VII
Sec 11.
o
The HR had issued a resolution recognizing GMA as the President. Both houses of
Congress started sending bills to be signed into law by GMA as President.
o
Both houses have recognized GMA as the President, implicitly clear in that recognition is
the premise that the inability of Erap is no longer temporary. Congress has rejected
Eraps claim of inability.
Issue of immunity: The Court rejects his argument that he cannot be prosecuted for the reason
that he must first be convicted in the impeachment proceedings. The impeachment trial of
petitioner Estrada was aborted by the walkout of the prosecutors and by the events that led to
his loss of the presidency. Since, the Impeachment Court is now functus officio, it is untenable for
petitioner to demand that he should first be impeached and then convicted before he can be
prosecuted.
o
The scope of immunity that can be claimed by Erap as a non-sitting President. The cases
filed against Erap are criminal in character. They involve plunder, bribery and graft and
corruption. By no stretch of the imagination can these crimes, especially plunder which
carries the death penalty, be covered by the alleged mantle of immunity of a non-sitting
president.

Updated Champ Digests

Fel. Jill. Lauren. Mon. Tara. Charms

POLITICAL LAW REVIEW


Atty. Jacinto Jimenez
SECTION 13

Civil Liberties Union vs. ES


prohibition against holding another office or employment

Petitioners assail constitutionality of EO 284: allowing members of cabinet, Usecs, Asecs to hold other
government offices in addition to their primary positions. They contend that this runs counter to Sec
13 which provides that members of Cabinet, deputies, assistants, shall not hold any other office or
employment during their tenure
EO 284: allows these people to hold not more than 2 positions.
The Secretary of Justice Ordonez, issued an opinion stating that the members of Cabinet may hold
another position if allowed by the primary functions of their respective positions. (Thus, this is the
legal basis relied upon in issuing EO 284)
The petitioners further argue that the E0284 adds to the exceptions under Sec 13. They contend that
the phrase unless otherwise provided by this constitution, should refer only to the following
exceptions:
o
1) the VP may be appointed as member of cabinet
o
2) the Sec of DOJ is an ex-officio member of the JBC.
Petitioners further argue that the exception under Article IX-B Sec 7(2) on the Civil Service
Commission applies only to officers and employees of the civil service, and cannot extend to Article
VII Sec 13 (the Cabinet, etc)

ISSUE: Does the prohibition under Sec 13 (on the prohibition against Cabinet members) admit exceptions
made for appointive officials under Article IX-B Sec 7(2)
SC: NO.
Although Article IX-B Sec 7(2) already contains a blanket prohibition against the holding of multiple offices
or employment in government, the framers of the Constitution saw it fit to FORMULATE A SIMILAR
PROVISION under Article VII, specifically prohibiting the President, VP, members of Cabinet from holding
any other office or employment during their tenure.
Evidently, the intent was to IMPOSE A STRICTER PROHIBITION on the President and his official family.
In fact, the wording of Sec 13 in Article VII is stricter. The disqualification is ABSOLUTE because there are
no qualifications made. The prohibition under Article VII is all-embracing and covers BOTH PUBLIC AND
PRIVATE office or employment.
Looking closer into Sec 13, second sentence again there is a sweeping, all-embracing prohibition
imposed on the President and members of Cabinet, which prohibitions are not similarly imposed on other
public officials, nor members of Congress, nor members of civil service, AFP. This is another proof of the
intent of the framers to treat the President and members of his official family A CLASS IN ITSELF.
Commissioner Foz stated: We actually have to be stricter with the President and members of Cabinet
because they exercise more powers, and therefore more checks and restraints on them are called for
because there is more possiblilty of abuse.
In short, Article IX-B Sec 7(2) lays the general rule applicable to both elective and appointive officals.
Article VII Sec 13 on the other hand is meant to be the exception applicable only to the President, VP,
Members of Cabinet.
BUT the prohibition against holding dual or multiple offices must not be construed as applying to posts
occupied by these executive officials
1) WITHOUT ADDITIONAL COMPENSATION
2) IN AN EX-OFFICIO CAPACITY,
3) AS PROVIDED FOR BY LAW AND
4) AS REQUIRED BY TH PRIMARY FUNCTIONS OF SAID OFFICIALS OFFICE.
(meaning they can hold another office)
The reason is that these posts do not comprise any other office since they are MERELY AN IMPOSITION
OF ADDITIONAL DUTIES AND FUNCTIONS.
For example, it would be absurd if the President cannot be the chairman of the NSC.
The term ex-officio means from office or by virtue of office.
Updated Champ Digests

Fel. Jill. Lauren. Mon. Tara. Charms

POLITICAL LAW REVIEW


Atty. Jacinto Jimenez

It refers to an authority derived from official character, not expressly conferred upon the individual, but
rather annexed to the official position.
It also means any act done in official character, or as a consequence of office, and without any other
appointment than that conferred by the office.
Note also that the additional duties must not only be closely related to , but must be required by the
officials primary functions. If the functions required to be performed are merely incidental, remotely
related, inconsistent, incompatible, or alien to the primary function, then it would be prohibited.
Because ex-officio position is really part of the principal office, then it follows that there would be no right
to receive additional compensation. These services are already paid for and covered by the compensation
from his principal office. (not entitled to per diem, honorarium, allowance, or any other euphemism)
LAW IS NULL AND VOID.

SECTION 15

IN RE: VALENZUELA
prohibition against midnight appointments

On March 30, the president appointed Valenzuela and Vallarta as RTC judges.
Also, appointments to the CA have been made, dated March 11, 1998 (the day before the
constitutional ban on appointments)
May 4, 1998 the Chief Justice received a letter from the President requesting for the final list of
nominees so that the vacancy could be filled up within 90 days from the time the vacancy occurred
(Article VIII Sec 4).
DOJ Secretary Bello requested a GUIDANCE from the Chief Justice, regarding the possibility of
holding an immediate meeting of the JBC to fill up the vacancy.
The Chief Justice replied saying that there are no sessions scheduled for the JBC until after the May
11 1998 elections. And so they still need to undertake a further study of the matter. The Chief Justice
also said that the list of nominees will follow after the elections. (The CJ convenes the JBC.)
However, it appears that the Justice Secretary Bello and some members of the JBC held a secret
meeting, and came to an agreement that if the CJ still does not call or convene the JBC, then they will
be constrained to convene in order to fill up the vacancy. And so, Secretary Bello and the others met
regardless of the CJs wishes.
The CJ later received a letter from the President saying that the election-ban provision on Sec 15
applies only to executive appointments or appointments in the executive branch (not the judiciary).
The President also reiterated his request for the final list of nominees

ISSUEs:
How is the requirement of filing up vacancies in the judiciary to be interpreted?
Does the prohibition in Sec 15 come into play? Is the president prohibited from making any appointments?
SC:
During the period stated in Sec 15, the President is neither required nor allowed to make appointments to
the courts. This means that Article VIII Sec 4 simply means that the President is required to fill vacancies
in the courts within the time frame provided UNLESS PROHIBITED BY SECTION 15.
The provision that the vacancy shall be filled within 90 days contrasts with the prohibition under Sec 15,
which is couched in stronger negative language that the President shall not make appointments.
Sec 15 is directed against 2 types of appointments:
1) those made for buying votes refers to appmts made w/in 2 mos prior to election
2) those made for partisan considerations refers to midnight appmts.
Sec 15 is thus broader. It contemplates not only midnight appointments but also appointments presumed
made for the purpose of influencing the outcome of the Presidential elections.

Updated Champ Digests

Fel. Jill. Lauren. Mon. Tara. Charms

POLITICAL LAW REVIEW


Atty. Jacinto Jimenez
Sec 15 allows only a narrow exception, that is, making TEMPORARY APPOINTMENTS to EXECUTIVE
POSITOINS when continued vacancies would PREJUDICE PUBLIC SERVICE OR ENDANGER PUBLIC SAFETY.
The appointments of Valenzuela and Vallarta on March 30, 1998 obviously made during the period of the
ban. Thus they come within the first prohibition relating to appointments which are considered to be for
the purpose of buying votes or influencing the election.
While the filling of vacancies in the judiciary is undoubtedly in the public interest, there is not showing of
any compelling reason to justify the making of the appointment during the period of the ban. In fact,
there is a strong public policy for the prohibition against appointments made within the ban period.
RECAP:
General Rule: Sec 15 Article VII (2-mos ban) prevails over Sec 4 of Article VIII (need to fill up vacancy 90days) during the period of the ban which is 2 months before the Presidential elections, the President is
not required to make appointments to fill up vacancies in the judiciary.
Exceptions: temporary appointments to executive when required by public service or public safety
Note also, this situation arises only once every 6 years!!

SECTION 16

RAFAEL VS. EMBROIDERY


appointments and the appointing power

Rafael operates a manufacturing bonded warehouse.


RA 3137 was enacted creating the Embroidery and Apparel Board to control and inspect the tax free
raw materials imported in the Phils)
The law provides for the composition of the Board:
o
1 rep from the Bureau of Customso
1 rep from the Central Bank
o
1 rep from the DTI
o
1 rep from the National Economic Council
o
1 rep from the private sector which shall come from the Association of Embroidery and
Apparel exporters of the Phils.
Rafael assails the constitutionality of the law, specifically the composition of the Board. He argues that
while Congress may create an office, it cannot specify who shall be appointed therein and that the
members of the Board should be appointed by the President. He contends that the law infringes on
the power of the President to make appointments.

ISSUE: Is the law valid?


SC: VALID.
It will be noted that for the chairman and the members of the Board to qualify, they need only be
designated by the respective department heads. THEY ALL SIT EX-OFFICIO, except the 1 rep from the
private sector. In order to be designated they must already be holding positions in the office mentioned in
the law. Thus, no new appointments are necessary. These first 4 offices perform functions which have
direct relation to the importation of raw materials, the manufacture thereof and their exportation abroad.
Thus, these representatives so designated merely perform duties in the Board in addition to those they
already perform under their original appointments. There is no attempt to deprive the President of his
power to make appointments.
Thus, their tenure on the Board is merely ON DETAIL, SUBJECT TO RECALL BY THEIR RESPECTIVE
CHIEFS.
The law does not violate the rule that the appointing power is the exclusive prerogative of the President
which no limitations may be imposed by Congress.

Updated Champ Digests

Fel. Jill. Lauren. Mon. Tara. Charms

POLITICAL LAW REVIEW


Atty. Jacinto Jimenez

BERMUDEZ VS. TORRES


nature of appointing power

There was a vacancy for the post of Provincial Prosecutor of Tarlac


Bermudez was the 1st Asst Provincial Prosecutor of Tarlac, who was a recommended by DOJ Sec.
Guingona.
Quiaoit was the one recommended by Congressman Jose Yap.
FVR appointed Quiaoit. Quiaoit assumed office.
Bermudez refused to vacate the office claiming that the original copy of Quiaoits appointment had not
yet been released by the DOJ Secretary.
Bermudez and Quiaoit were both called to the office of DOJ Secretary. Bermudez was ordered to wind
up his cases and turn-over the office to Quiaoit.
Thus, Bermudez challenged the appointment of Quiaoit primarily on the ground that the appointment
lacks the recommendation of the DOJ Sec. Bermudez relies on the Revised Admin Code stating that
prosecutors shall be appointed by the President upon the recommendation of the DOJ Sec.

ISSUE: Is the recommendation of the DOJ Secretary absolutely essential to the appointment of Quiaoit as
Prosecutor?
SC: ABSOLUTELY NO.
An appointment to public office is the unequivocal act of designating or selecting by one having the
authority therefor of an individual to discharge and perform the duties and functions of an office or trust.
APPOINTMENT NECESSARILY CALLS FOR AN EXERCISE OF DISCRETION ON THE PART OF THE
APPOINTING POWER.
The power to appoint is essentially discretionary. The appointing power has the right of choice which he
may exercise freely according to his judgment, deciding for himself who is best qualified among those who
have the necessary qualifications and eligibilities. It is the prerogative of the appointing power. The RIGHT
OF CHOICE IS THE HEART OF THE POWER TO APPOINT.
The President, as HEAD OF GOVT has the power of control over all executive departments, bureaus and
offices. Control means the authority to alter or modify or even nullify or set aside what a subordinate
office has done, as well as to substitute the judgment of the latter, as when the former deems it to be
appropriate.
Hence, the President has the power TO ASSUME DIRECTLY the functions of an executive department,
bureau or office.
The recommendation of the DOJ Secretary should be interpreted to be a MERE ADVICE, EXHORTATION,
INDORSEMENT, which is essentially persuasive in character, NOT BINDING, NOR OBLIGATORY, upon the
President. The recommendation is nothing really more than advisory in nature.

SARMIENTO VS. MISON


APPOintments which do not need CA confirmation

Petitioners as taxpayers assail constitutionality of Misons appointment as Commisioner of Bureau of


Customs. They contend that the appointment has not been confirmed by the CA.
The Consti provides for 4 groups whom the President shall appoint:
1) head of executive departments, ambassadors, consuls, officers of the AFP with rank of colonel
or naval captain and above, other officers whose appointment are vested in him in this
Constitution:
a) members of the JBC,
b) Chairman / commissioners of CSC, Comelec, COA
c) members of regular consultative commissions.
2) all other officers of Government whose appointments are not otherwise provided for by law
3) those whom the President may be authorized by law to appoint.
4) officers lower in rank whose appointments the Congress may vest in the President alone
The first item is uncontested, they clearly require the consent of the CA.
The 2nd, 3rd, 4th items are the ones disputed.

Updated Champ Digests

Fel. Jill. Lauren. Mon. Tara. Charms

POLITICAL LAW REVIEW


Atty. Jacinto Jimenez

As to the 4th group, it is argued that since a law is needed to vest the appointment of lower-ranked
officers in the President alone, this implies that the absence of such law, a lower-ranked officer has to
be appointed by the President subject to confirmation by the CA. consequently, higher ranked officers
should be appointed by the President, also subject to confirmation by the CA

ISSUE: Does the position of Commissioner of BoC require CA confirmation?


SC:VALID APPOINTMENT
The 2nd, 3rd groups do not require CA confirmation. The power to appoint is fundamentally executive or
presidential in character. Limitations on or qualifications of such power should be strictly construed against
them. Such limitation or qualifications must be clearly stated in order to be recognized. It is only in the
first sentence of Sec 16where it is clearly stated that appointments by the President to the positions
enumerated therein require the consent of the CA.
As to the 4th group, the clear and express intent of the constitution was to exclude presidential
appointments from CA confirmation, except the appointments to offices expressly mentioned in the first
sentence of Sec 16.
Thus, in the case of lower-ranked officers, the Congress may by law vest their appointment in the
President, in the courts, or in the heads of various departments. In short the word, ALONE, appears to
be redundant.
It is thus evident that the position of COMMISSIONER OF THE BOC, (a bureau head), is NOT ONE OF
THOSE WITHIN THE FIRST GROUP of appointments where consent of the CA is required. As already said,
the constitution deliberately excluded the position of bureau heads from the appointments that need the
CA confirmation.
Moreover, the President is expressly authorized by law to appoint the Commissioner of the BOC, under Sec
601 of the RA 1937.

CALDERON VS. CARALE


appointive positions subject to CA confirmation

RA 6715 (the Herrera Veloso Law) was enacted to amend the Labor Code. It provided that the
Chairman and Commissioners of the NLRC shall be appointed by the President and subject to CA
confirmation
President Aquino thus appointed the Chairman and Commissioners of NLRC.
This was challenged by Calderon who assails the legality of the permanent appointments made by the
President to the respondent Chairman and Commissioners of the NLRC.
Calderon claims that RA 6715 must be complied with since Congress may by law, require confirmation
by the CA on other officers aside from those mentioned in the Constitution. He claims that the rulings
in Mison and Bautista are not relevant here because there is a law RA 6715 which requires CA
confirmation of such appointments.
Solgen on the other hand contends that RA 6715 unduly expands the confirmation powers of the CA.

ISSUE: Whether Congress can, by law, require CA confirmation of appointments made by the President, in
addition to those expressly mentioned in Sec 16.
SC: NO. UNCONSTITUTIONAL.
The NLRC officers fall within the 2nd sentence of Sec 16, or the 3rd group referred to in the Mison ruling
(those whom the President may be authorized by law to appoint).
The requirement of CA confirmation on appointments of the officers of the NLRC is unconstitutional
because
1) it amends by legislation the first sentence of the Sec 16 of the Constitution, adding thereto
appointments requiring CA confirmation.
2) it amends by legislation the second sentence of Sec 16 of the Constitution, by imposing the
confirmation of the CA on appointments which are otherwise entrusted only to the President.
Again, the confirmation of the CA is required only for presidential appointments mentioned in the first
sentence of Sec 16 (or the first group in the Mison ruling). Under the second sentence, the President
maybe authorized to appoint without need for CA confirmation.
Updated Champ Digests

Fel. Jill. Lauren. Mon. Tara. Charms

POLITICAL LAW REVIEW


Atty. Jacinto Jimenez

Chairman and Members of NLRC NO NEED FOR CA CONFIRMATION.

RUFINO V. ENDRIGA, July 21, 2006


Article VII, Section 16

Presidential Decree No. 15 (PD 15) created the Cultural Center of the Philippines (CCP) for the
primary purpose of propagating arts and culture in the Philippines. PD 15 increased the members
of CCP's Board from seven to nine trustees. Later, Executive Order No. 1058, increased further
the trustees to 11.

Eventually, during the term of Ramos, the CCP Board included the Endriga Group

the Endriga group filed a petition for quo warranto questioning Estrada's appointment of seven
new members to the CCP Board. They claimed that it is only when the CCP Board is entirely
vacant may the President of the Philippines fill such vacancies, acting in consultation with the
ranking officers of the CCP.
o
The clear and categorical language of Section 6(b) of PD 15 states that vacancies in the
CCP Board shall be filled by a majority vote of the remaining trustees. Should only
one trustee survive, the vacancies shall be filled by the surviving trustee acting in
consultation with the ranking officers of the CCP. Should the Board become
entirely vacant, the vacancies shall be filled by the President of the Philippines
acting in consultation with the same ranking officers of the CCP. Thus, the remaining
trustees, whether one or more, elect their fellow trustees for a fixed four-year term. On
the other hand, Section 6(c) of PD 15 does not allow trustees to reelect fellow trustees
for more than two consecutive terms.
o
The Endriga group asserted that when former President Estrada appointed the Rufino
group, only one seat was vacant due to the expiration of Maosa's term. The CCP Board
then had 10 incumbent trustees. They maintained that under the CCP Charter, the
trustees' fixed four-year term could only be terminated "by reason of resignation,
incapacity, death, or other cause." Presidential action was neither necessary nor justified
since the CCP Board then still had 10 incumbent trustees who had the statutory power
to fill by election any vacancy in the Board.
o
The Endriga group refused to accept that the CCP was under the supervision and control
of the President. The Endriga group cited Section 3 of PD 15, which states that the CCP
"shall enjoy autonomy of policy and operation x x x."

Estrada appointed seven new trustees to the CCP Board for a term of four years to replace the
Endriga group as well as two other incumbent trustees. The Rufino group took their oaths of
office and assumed the performance of their duties.

Rufino Group: that the law could only delegate to the CCP Board the power to appoint officers
lower in rank than the trustees of the Board. Section 6(b) of PD 15 authorizing the CCP trustees
to elect their fellow trustees should be declared unconstitutional being repugnant to Section 16,
Article VII of the 1987 Constitution allowing the appointment only of "officers lower in rank" than
the appointing power.
CA: Endriga group entitled to the office.

Issue: w/n Section 6(b) of PD 15 is unconstitutional considering that:


A. [it] is an invalid delegation of the President's appointing power under the Constitution;
B. [it] effectively deprives the President of his constitutional power of control and supervision over the CCP
Held: UNCONSTITIONAL
POWER TO APPOINT

The source of the President's power to appoint, as well as the Legislature's authority to delegate
the power to appoint, is found in Section 16, Article VII of the 1987 Constitution which provides:
the President shall nominate and, with the consent of the Commission on Appointments, appoint
the heads of the executive departments, ambassadors, other public ministers and consuls, or
officers of the armed forces from the rank of colonel or naval captain, and other officers whose
appointments are vested in him in this Constitution. He shall also appoint all other officers of the
Government whose appointments are not otherwise provided for by law, and those whom he may
be authorized by law to appoint. The Congress may, by law, vest the appointment of other
officers lower in rank in the President alone, in the courts, or in the heads of
departments, agencies, commissions, or boards.The President shall have the power to
make appointments during the recess of the Congress, whether voluntary or compulsory, but

Updated Champ Digests

Fel. Jill. Lauren. Mon. Tara. Charms

POLITICAL LAW REVIEW


Atty. Jacinto Jimenez
such appointments shall be effective only until disapproval by the Commission on Appointments
or until the next adjournment of the Congress. (Emphasis supplied)

The power to appoint is the prerogative of the President, except in those instances when the
Constitution provides otherwise. Usurpation of this fundamentally Executive power by the
Legislative and Judicial branches violates the system of separation of powers that inheres in our
democratic republican government.
Under Section 16, Article VII of the 1987 Constitution, the President appoints three groups of
officers.
1. heads of the Executive departments, ambassadors, other public ministers and consuls,
officers of the armed forces from the rank of colonel or naval captain, and other officers
whose appointments are vested in the President by the Constitution.
w/ the
Commission of Appointments consent
2. those whom the President may be authorized by law to appoint. consent not required
3. all other officers of the Government whose appointments are not otherwise provided by
law. consent not required

appoints the third group of officers if the law is silent on who is the appointing
power, or if the law authorizing the head of a department, agency, commission,
or board to appoint is declared unconstitutional. Thus, if Section 6(b) and (c) of
PD 15 is found unconstitutional, the President shall appoint the trustees of the
CCP Board because the trustees fall under the third group of officers.
o
* there is a fourth group of lower-ranked officers whose appointments Congress may by
law vest in the heads of departments, agencies, commissions, or boards.
The grant of the power to appoint to the heads of agencies, commissions, or boards is a
matter of legislative grace. Congress has the discretion to grant to, or withhold from, the
heads of agencies, commissions, or boards the power to appoint lower-ranked officers. If it so
grants, Congress may impose certain conditions for the exercise of such legislative delegation,
like requiring the recommendation of subordinate officers or the concurrence of the other
members of the commission or board.

This is in contrast to the President's power to appoint which is a self-executing power


vested by the Constitution itself and thus not subject to legislative limitations or conditions. 28 The
power to appoint conferred directly by the Constitution on the Supreme Court en banc29 and on
the Constitutional Commissions 30 is also self-executing and not subject to legislative limitations or
conditions.

The framers of the 1987 Constitution clearly intended that Congress could by law vest the
appointment of lower-ranked officers in the heads of departments, agencies, commissions, or
boards. these inferior or lower in rank officers are the subordinates of the heads of
departments, agencies, commissions, or boards who are vested by law with the power
to appoint. The express language of the Constitution and the clear intent of its framers point to
only one conclusion the officers whom the heads of departments, agencies, commissions, or
boards may appoint must be of lower rank than those vested by law with the power to appoint.

Also, the power to appoint can only be vested in the HEADS of the named offices. The word
"heads" refers to the chairpersons of the commissions or boards and NOT TO THEIR MEMBERS,
for several reasons:
o
a plain reading of the last sentence of the first paragraph of Section 16, Article VII of the
1987 Constitution shows that the word "heads" refers to all the offices succeeding that
term, namely, the departments, agencies, commissions, or boards. This plain reading is
consistent with other related provisions of the Constitution.
o
agencies, like departments, have no collegial governing bodies but have only chief
executives or heads of agencies. Thus, the word "heads" applies to agencies. Any other
interpretation is untenable.
o
all commissions or boards have chief executives who are their heads. Since the
Constitution speaks of "heads" of offices, and all commissions or boards have chief
executives or heads, the word "heads" could only refer to the chief executives or heads
of the commissions or boards.
o
the counterpart provisions of Section 16, Article VII of the 1987 Constitution in the 1935
and 1973 Constitutions uniformly refer to "heads" of offices. The 1935 Constitution
limited the grant of the appointment power only to "heads of departments." 32 The 1973
Constitution expanded such grant to other officers, namely, "members of the Cabinet, x
x x, courts, heads of agencies, commissions, and boards x x x." 33 If the 1973
Constitution intended to extend the grant to members of commissions or boards, it could
have followed the same language used for "members of the Cabinet" so as to state
"members of commissions or boards." Alternatively, the 1973 Constitution could have

Updated Champ Digests

Fel. Jill. Lauren. Mon. Tara. Charms

POLITICAL LAW REVIEW


Atty. Jacinto Jimenez

placed the words commissions and boards after the word "courts" so as to state
"members of the Cabinet, x x x, courts, commissions and boards." Instead, the 1973
Constitution used "heads of agencies, commissions, and boards."
the 1935, 1973, and 1987 Constitutions make a clear distinction whenever granting the
power to appoint lower-ranked officers to members of a collegial body or to the head of
that collegial body. Thus, the 1935 Constitution speaks of vesting the power to appoint
"in the courts, or in the heads of departments." Similarly, the 1973 Constitution speaks
of "members of the Cabinet, courts, heads of agencies, commissions, and boards."
As an enumeration of offices, what applies to the first office in the enumeration also
applies to the succeeding offices mentioned in the enumeration. Since the words " in the
heads of" refer to "departments," the same words "in the heads of" also refer to the
other offices listed in the enumeration, namely, "agencies, commissions, or boards."

Thus, the Chairman of the CCP Board is the "head" of the CCP who may be vested by law, under
Section 16, Article VII of the 1987 Constitution, with the power to appoint lower-ranked officers
of the CCP.

the CCP is a public corporation governed by a Board of Trustees. The CCP, being governed by a
board, is not an agency but a board for purposes of Section 16, Article VII of the 1987
Constitution.

** Section 6(b) and (c) of PD 15 is thus irreconcilably inconsistent with Section 16, Article VII of
the 1987 Constitution. Section 6(b) and (c) of PD 15 empowers the remaining trustees of the CCP
Board to fill vacancies in the CCP Board, allowing them to elect their fellow trustees. On the other
hand, Section 16, Article VII of the 1987 Constitution allows heads of departments, agencies,
commissions, or boards to appoint only "officers lower in rank" than such "heads of
departments, agencies, commissions, or boards." This excludes a situation where the appointing
officer appoints an officer equal in rank as him. Thus, insofar as it authorizes the trustees of the
CCP Board to elect their co-trustees, Section 6(b) and (c) of PD 15 is unconstitutional because it
violates Section 16, Article VII of the 1987 Constitution.
o
It does not matter that Section 6(b) of PD 15 empowers the remaining trustees to
"elect" and not "appoint" their fellow trustees for the effect is the same, which is to fill
vacancies in the CCP Board. A statute cannot circumvent the constitutional limitations on
the power to appoint by filling vacancies in a public office through election by the coworkers in that office. Such manner of filling vacancies in a public office has no
constitutional basis.

Further, Section 6(b) and (c) of PD 15 makes the CCP trustees the independent appointing power
of their fellow trustees. The creation of an independent appointing power inherently conflicts with
the President's power to appoint. This inherent conflict has spawned recurring controversies in
the appointment of CCP trustees every time a new President assumes office.

POWER OF CONTROL OVER THE EXECUTIVE BRANCH

The presidential power of control over the Executive branch of government extends to all
executive employees from the Department Secretary to the lowliest clerk. 35 This constitutional
power of the President is self-executing and does not require any implementing law. Congress
cannot limit or curtail the President's power of control over the Executive branch. 36

The CCP falls under the Executive branch. Since the President exercises control over "all the
executive departments, bureaus, and offices," the President necessarily exercises control over the
CCP which is an office in the Executive branch. In mandating that the President "shall have
control of all executive x x x offices," Section 17, Article VII of the 1987 Constitution does not
exempt any executive office one performing executive functions outside of the independent
constitutional bodies from the President's power of control. There is no dispute that the CCP
performs executive, and not legislative, judicial, or quasi-judicial functions.

The Legislature cannot validly enact a law that puts a government office in the Executive branch
outside the control of the President in the guise of insulating that office from politics or making it
independent. If the office is part of the Executive branch, it must remain subject to the control of
the President. Otherwise, the Legislature can deprive the President of his constitutional power of
control over "all the executive x x x offices." If the Legislature can do this with the Executive
branch, then the Legislature can also deal a similar blow to the Judicial branch by enacting a law
putting decisions of certain lower courts beyond the review power of the Supreme Court. This will
destroy the system of checks and balances finely structured in the 1987 Constitution among the
Executive, Legislative, and Judicial branches.

Section 6(b) and (c) of PD 15, which authorizes the trustees of the CCP Board to fill vacancies in
the Board, runs afoul with the President's power of control under Section 17, Article VII of the

Updated Champ Digests

Fel. Jill. Lauren. Mon. Tara. Charms

POLITICAL LAW REVIEW


Atty. Jacinto Jimenez
1987 Constitution. The intent of Section 6(b) and (c) of PD 15 is to insulate the CCP from political
influence and pressure, specifically from the President. 44 Section 6(b) and (c) of PD 15 makes the
CCP a self-perpetuating entity, virtually outside the control of the President. Such a public office
or board cannot legally exist under the 1987 Constitution.
TINGA dissents
SECTION 17
ANG-ANGCO VS. CASTILLO
power of control and security of tenure

Pepsi sent a letter to the Secretary of Commerce to request for a special permit to withdraw pepsi
concentrates from the customs house. They were imported without any dollar allocation or forex
remittance.
They also wrote the Secretary of Finance, the Central Bank Governor.
The Import-Export Committee of the CB submitted to the Monetary Board a memorandum of Pepsis
letter. The Monetary Board however failed to take up the matter because the transaction did not
involve any dollar allocation or forex.
Pepsi still had no authorization to withdraw the concentrates.
So, Pespsi approached Angangco (the Collector of Customs) to secure the immediate release of the
concentrates.
However, since there was no CB certificate, Angangco told Pepsi to get one from the No-Dollar Import
Office who had jurisdiction over the case.
Later, the said Office also said that it was not within their jurisdiction.
Thus, Angangco called up the Secretary of Finance (Hernandez) and the Secretary VERBALLY
APPROVED THE RELEASE OF THE PEPSI PRODUCTS.
Angangco thus authorized the release.
The Commissioner of Customs Manahan however ordered the seizure of the goods. Manahan filed a
case against Angangco for grave neglect of duty, prejudicial to the Bureau of Customs
President Magsaysay wanted an investigation. Angangco was suspended from office but was
reinstated by Secretary of Finance Hernandez.
Magsaysay died.. But after 3 years Executive Secretary Castillo by authority of now President Garcia,
rendered a decision finding Angangco GUILTY and CONSIDERED RESIGNED.
Angangco assails this decision and argues that it deprives him of his office without due process.
Castillo however claims that the President has the power of control over officers and employees in the
executive department.

ISSUE: Whether the President has the power to take direct action even if Angangco belongs to the
classified service under Civil Service Act of 1959
SC: Angangco was deprived of due process!!
Officers in the CLASSIFIED CIVIL SERVICE comes within the EXCLUSIVE JURISDICTION OF THE
COMMISSION OF CIVIL SERVICE, except as otherwise provided for by law. So is there any other law
empowering the President to remove officers in the classified civil service?
The only law we can recall is the RAC which grants the power to remove officials conformably to law. It
shows that the President does not have blanket authority to remove any officer or employee of the
government, but that his power must still be subject to the law that may be passed. Here, there is such a
law (Civil Service ACT) which governs officers and employees in the classified civil service. The President is
bound to follow that law.
So, even granting for administrative purpose the President is considered the Department Head of the CSC,
his power to remove is still subject to the Civil Service Act. Under said law, the proper forum would be the
COMMISSIONER OF THE CIVIL SERVICE, OR THE CIVIL SERVICE APPEALS BOARD.
As to the power of control, it is defined as the power of an officer to alter or modify or nullify or set aside
what a subordinate officer had done in the performance of his duties and to substitute the judgment of
the former for that of the latter.
The Presidents control over the executive department REFERS ONLY TO MATTERS OF GENERAL POLICY.
Policy means any settled or definite course or method adopted and followed by a government or body.

Updated Champ Digests

Fel. Jill. Lauren. Mon. Tara. Charms

POLITICAL LAW REVIEW


Atty. Jacinto Jimenez

Here, the REMOVAL OF AN OFFICER (Angangco) cannot be said to come within the meaning of CONTROL
OVER A SPECIFIC POLICY OF GOVERNMENT.
The power of control of the President may indeed extend to the power to investigate, suspend or remove
officers who belong to the executive, under the principle that the power to remove is inherent in the
power to appoint. However this applies only if they are presidential appointees or do not belong to the
classified service.
As to those officers who belong to the classified service that power cannot be exercised.
The action of the Executive Secretary, without submitting the case to the Commissioner of Civil Service, is
contrary to law.
Angangco reinstated.
VILLALUZ VS. ZALDIVAR
power to remove appointee

Villaluz was the Motor Vehicles Office Adminstrator. He was removed on allegations of graft and
corruption.
In a letter to the President, Congressman Joaquin Roces outlined the alleged gross mismanagement
and inefficiency committed by Villaluz at the Office. Congressman Roces recommended the
replacement of Villaluz.
Executive Secretary Castillo thus suspended Villaluz. (He was earlier given 72 hours to explain his
side)
The ES also created an investigating committee. The committee submitted its report to the President.
The President later issued AO # 332, removing Villaluz from office.
Villaluz assails this order and claims that he was denied due process. He also claimed that he is under
the control and supervision of the DPWH and under the jurisdiction of the CSC.

ISSUE: Does the President have jurisdiction to remove Villaluz from office?
SC: YES.
Villaluz is a PRESIDENTIAL APPOINTEE, as such he belongs to the non-competitive or unclassified service.
He can only be investigated and removed from office after due hearing by the President under the
principle that the power to remove is inherent in the power to appoint.
Hence, the CSC is without jurisdiction to hear and decide the administrative charges against Villaluz
because the authority of the CSC can only be exercised with reference to permanent officials and
employees in the classified service.
Even if the petitioner is under the control and supervision of the DPWH, since he is still a presidential
appointee, he comes exclusively within the jurisdiction of the President.
Also, even if the charges made by Congressman Roces were not sworn to, the proceedings are still valid
since the President as Chief Executive, and as Administrative Head, may motu proprio commence
administrative proceedings without need of a verified complaint.

NAMARCO VS. ARCA


power of control over GOCC

Juan Arrive was Manager at the Storage Department of NAMARCO. He was investigated for violation
of memorandum order regarding the improper release of shipment. The investigating committee
found him GUILTY and the general manager of Namarco, including the Board of Directors, dismissed
him.
Arrive appealed to the President. The Exec Secretary (acting for the President) set aside the Namarco
ruling, and REINSTATED ARIVE.
Namarco now assails the actions of the ES and contended that the President had no jurisdiction to
reverse its decision.
The constitution provides that the President has control over bureaus and offices.

Updated Champ Digests

Fel. Jill. Lauren. Mon. Tara. Charms

POLITICAL LAW REVIEW


Atty. Jacinto Jimenez

Namarco claims that the word OFFICES should refer only to offices performing governmental
functions, those without separate juridical personality. Namarco claims that GOCCs like Namarco,
should not be included in the term Offices
Namarco further claims that the Namarco Charter grants only the General Manager and the Board of
Directors the power to remove employees.

ISSUE: Whether the President has the power to review the decisions of GOCCs?
SC: YES.
We hold that the President has the authority to review and reverse the decision of Namarco. It falls within
the constitutional power of the President over all executive departments, bureaus and offices.
A GOCC such as Namarco, partakes the nature of government bureau or office, which are administratively
supervised by the Administrator of the Office of Economic Coordination, whose rank is equivalent to a
head of an Executive Department, and is responsible to the President under whose control his functions
shall be exercised.
The right to appeal to the President reposes upon the President the power of control over executive
departments. Under the PRESIDENTIAL SYSTEM OF GOVERNMENT, all all executive and administrative
organization are adjuncts of the Executive Department. The heads of the various executive departments
are agents of the Chief Executive.
Order of Reinstatement VALID.

SECTION 18

LANSANG vs. GARCIA


suspension of privilege of writ

There was this Plaza Miranda Bombing on August 21 1979. The Liberal Party was having a campaign
for the elections. President Marcos issued Proclamation 889 suspending the privilege of writ of habeas
corpus.
Lansang et al were arrested without a warrant, on reasonable belief that they had participated in the
crime of insurrection or rebellion.
They now file this petition for writ of habeas corpus and assail the validity of Proclamation 889.
Garcia, the head of the PC, claims that their continued suspension is valid due to the suspension of
the privilege, and because there was a state of insurrection or rebellion in this country.

ISSUE: Since the authority to decide whether an exigency has arisen requiring the suspension of the
privilege belongs to the President, does the Court have the power to review this decision?
SC: YES. The Court has authority to inquire into the existence of the factual bases for the suspension of
the privilege.
ISSUE: Petitioners claim that Proclamation 889 never declared the existence of actual invasion
insurrection or rebellion, nor immediate danger thereof. They claim that there was no factual basis for the
Proclamation.
SC: This has been rendered MOOT because a subsequent Proclamation 889-A was issued stating that
there were lawless elements who have entered into a conspiracy. And are actually engaged in armed
insurrection and rebellion. In order to seize power Hence, there was a declaration of the existence of
actual conspiracy and of the intent to rise in arms.
ISSUE: Is the Proclamation conclusive upon the courts?
SC: NO.
Two conditions must occur for the valid exercise of authority to suspend the privilege of writ of habeas
corpus.
1) invasion, insurrection, rebellion,
2) public safety requires such suspension.

Updated Champ Digests

Fel. Jill. Lauren. Mon. Tara. Charms

POLITICAL LAW REVIEW


Atty. Jacinto Jimenez

In this case, both are declared in the Proclamation, however the Court still has the authority to inquire
into the existence of the factual bases in order to determine the constitutionality thereof.
The grant of the power to suspend the privilege is neither absolute nor unqualified. The authority is limited
and conditional. The authority to suspend the privilege is thus circumscribed, confined and restricted not
only by the PRESCRIBED SETTINGS or the CONDITIONS essential to its existence, but also as regards the
TIME AND PLACE where it may be exercised.
ISSUE: Is the Proclamation valid? Does it have basis.
SC: YES.
There exists a rebellion even if the NPA is too small compared to the AFP. Size does not matter. It does not
negate the existence of rebellion. Rebellion need not be widespread or attain the magnitude of a civil war.
The crime of rebellion may be limited in its scope to any part in the Philippines. In short, there was
rebellion.
If the courts no longer functioned, then suspension would not be necessary, there being no courts to issue
the writ. (In short, even if there are normal operations of the courts, this does not indicate that the
Proclamation was invalid).
ISSUE: Can the President suspend the privilege in the entire country?
SC: YES he can.
He needed some time to find out how the suspension works, so he caused the suspension to be gradually
lifted in some provinces thereafter.
**Note that the President could have declared a general suspension of the privilege.
Instead, Proclamation 889 limited the suspension to persons detained for crimes of insurrection and
rebellion only. P-889-A further limited the coverage.
Under the Constituion, the President had 3 options:
1) to call out the Armed Forces
2) to suspend the privilege of writ of habeas corpus
3) to place the Philippines (entirely or partly) under martial law.
Marcos already called out the armed forces but was inadequate. Hence he chose the suspend the
privilege.
PD 889 VALID!
ABERCA VS. VER

Illegal searches and seizures and other human rights violations were alleged to have been committed
by the Task Force Makabansa of the AFP. General Fabian Ver ordered the pre-emptive strikes against
communist terrorists.
The victims now file an action for damages.
The AFP generals filed their MTD on the ground that judicial action is barred since the suspension of
the privilege of writ of habeas corpus precluded judicial inquiry into the legality of the detention.
RTC granted the MTD.
Petitioners appealed claiming that Art 32 of the Civil Code entitles them to damages.
Respondents however insist that they are covered by the mantle of state immunity for acts done in
the performance of official duties. They rely on Proclamation 2054, that as members of the AFP, they
were merely preventing or suppressing lawless violence, rebellion, subversion.
They further contend that the victims are circumventing the suspension by resorting to a civil suit
aimed at the same purpose of making judicial inquiry into the legality of their detention?

ISSUE: Does the suspension of the privilege bar a civil case? Are they immuned?
SC:
They are NOT IMMUNED FROM SUIT. The Proclamation cannot be a blanket license or a roving commission
untrammeled by any constitutional restraint, to disregard or transgress upon the rights and liberties of
individuals.

Updated Champ Digests

Fel. Jill. Lauren. Mon. Tara. Charms

POLITICAL LAW REVIEW


Atty. Jacinto Jimenez
The main relief they are asking is indemnification for alleged damages they suffered. Their cause of action
is inextricably linked on the same claim of violations of their constitutional rights that they invoked in the
habeas corpus case.
The suspension of the privilege does not destroy the petitioners right and cause of action for damages for
illegal arrest and detention and other violations of their consti rights. The SUSPENSION DOES NOT
RENDER VALID AN OTHERWISE INVALID ARREST OR DETENTION. What is suspended is merely the right
of the individual to seek release from detention through the writ of habeas corpus as a speedier means of
obtaining his liberty.
Effect of the Suspension: the court may not grant release of the persons detained but they court may
award damages.
David vs. Arroyo
Facts:
On February 24, 2006, as the nation celebrated the 20th Anniversary of the Edsa People Power I,
President Arroyo issued PP 1017 declaring a state of national emergency.
and in my capacity as their Commander-in-Chief, do hereby command the Armed Forces of the
Philippines, to maintain law and order throughout the Philippines, prevent or suppress all
forms of lawless violence as well as any act of insurrection or rebellion and to enforce
obedience to all the laws and to all decrees, orders and regulations promulgated by me
personally or upon my direction; and as provided in Section 17, Article 12 of the Constitution do
hereby declare a State of National Emergency.
On March 3, 2006, exactly one week after the declaration of a state of national emergency and after all
these petitions had been filed, the President lifted PP 1017. She issued Proclamation No. 1021.
In their presentation of the factual bases of PP 1017 and G.O. No. 5, respondents stated that the
proximate cause behind the executive issuances was the conspiracy among some military officers, leftist
insurgents of the New Peoples Army (NPA), and some members of the political opposition in a plot to
unseat or assassinate President Arroyo. 4 They considered the aim to oust or assassinate the President and
take-over the reigns of government as a clear and present danger.
During the oral arguments held on March 7, 2006, the Solicitor General specified the facts leading to the
issuance of PP 1017 and G.O. No. 5. Significantly, there was no refutation from petitioners
counsels.
SUBSTANTIVE ISSUES:
1) Whether the Supreme Court can review the factual bases of PP 1017.
As to how the Court may inquire into the Presidents exercise of power, Lansang adopted the test that
"judicial inquiry can go no further than to satisfy the Court not that the Presidents decision is correct," but
that "the President did not act arbitrarily." Thus, the standard laid down is not correctness, but
arbitrariness.83 In Integrated Bar of the Philippines, this Court further ruled that "it is incumbent upon
the petitioner to show that the Presidents decision is totally bereft of factual basis" and that if
he fails, by way of proof, to support his assertion, then "this Court cannot undertake an independent
investigation beyond the pleadings."
Petitioners failed to show that President Arroyos exercise of the calling-out power, by issuing PP 1017, is
totally bereft of factual basis. A reading of the Solicitor Generals Consolidated Comment and
Memorandum shows a detailed narration of the events leading to the issuance of PP 1017, with supporting
reports forming part of the records. Mentioned are the escape of the Magdalo Group, their audacious
threat of the Magdalo D-Day, the defections in the military, particularly in the Philippine Marines, and the
reproving statements from the communist leaders. There was also the Minutes of the Intelligence Report
and Security Group of the Philippine Army showing the growing alliance between the NPA and the military.
Petitioners presented nothing to refute such events. Thus, absent any contrary allegations, the Court is
convinced that the President was justified in issuing PP 1017 calling for military aid.
Indeed, judging the seriousness of the incidents, President Arroyo was not expected to simply fold her
arms and do nothing to prevent or suppress what she believed was lawless violence, invasion or rebellion.
However, the exercise of such power or duty must not stifle liberty.

Updated Champ Digests

Fel. Jill. Lauren. Mon. Tara. Charms

POLITICAL LAW REVIEW


Atty. Jacinto Jimenez
2) Whether PP 1017 and G.O. No. 5 are unconstitutional.
a. Facial Challenge
A facial review of PP 1017, using the overbreadth doctrine, is uncalled for.
First and foremost, the overbreadth doctrine is an analytical tool developed for testing "on their faces"
statutes in free speech cases, also known under the American Law as First Amendment cases.103
A plain reading of PP 1017 shows that it is not primarily directed to speech or even speech-related
conduct. It is actually a call upon the AFP to prevent or suppress all forms of lawless violence. In United
States v. Salerno,104 the US Supreme Court held that "we have not recognized an overbreadth
doctrine outside the limited context of the First Amendment" (freedom of speech).
econd, facial invalidation of laws is considered as "manifestly strong medicine," to be used "sparingly
and only as a last resort," and is "generally disfavored;"107 The reason for this is obvious. Embedded
in the traditional rules governing constitutional adjudication is the principle that a person to whom a law
may be applied will not be heard to challenge a law on the ground that it may conceivably be applied
unconstitutionally to others, i.e., in other situations not before the Court.
And third, a facial challenge on the ground of overbreadth is the most difficult challenge to mount
successfully, since the challenger must establish that there can be no instance when the assailed law
may be valid. Here, petitioners did not even attempt to show whether this situation exists.
Petitioners likewise seek a facial review of PP 1017 on the ground of vagueness. This, too, is unwarranted.
Related to the "overbreadth" doctrine is the "void for vagueness doctrine" which holds that "a law is
facially invalid if men of common intelligence must necessarily guess at its meaning and differ
as to its application."110 It is subject to the same principles governing overbreadth doctrine. For one, it is
also an analytical tool for testing "on their faces" statutes in free speech cases. And like overbreadth, it
is said that a litigant may challenge a statute on its face only if it is vague in all its possible
applications. Again, petitioners did not even attempt to show that PP 1017 is vague in all its
application. They also failed to establish that men of common intelligence cannot understand the
meaning and application of PP 1017.
b. Constitutional Basis
First provision:
"by virtue of the power vested upon me by Section 18, Artilce VII do hereby command the Armed
Forces of the Philippines, to maintain law and order throughout the Philippines, prevent or suppress all
forms of lawless violence as well any act of insurrection or rebellion"
Under the calling-out power, the President may summon the armed forces to aid him in suppressing
lawless violence, invasion and rebellion. This involves ordinary police action. But every act that goes
beyond the Presidents calling-out power is considered illegal or ultra vires. For this reason, a President
must be careful in the exercise of his powers. He cannot invoke a greater power when he wishes to act
under a lesser power. There lies the wisdom of our Constitution, the greater the power, the greater are the
limitations.
Some of the petitioners vehemently maintain that PP 1017 is actually a declaration of Martial Law. It is no
so. What defines the character of PP 1017 are its wordings. It is plain therein that what the President
invoked was her calling-out power.
Second Provision: "Take Care" Power
The second provision pertains to the power of the President to ensure that the laws be faithfully executed.
This is based on Section 17, Article VII which reads:

Updated Champ Digests

Fel. Jill. Lauren. Mon. Tara. Charms

POLITICAL LAW REVIEW


Atty. Jacinto Jimenez
SEC. 17. The President shall have control of all the executive departments, bureaus, and offices. He shall
ensure that the laws be faithfully executed.
As the Executive in whom the executive power is vested, 115 the primary function of the President is to
enforce the laws as well as to formulate policies to be embodied in existing laws. He sees to it that all laws
are enforced by the officials and employees of his department. Before assuming office, he is required to
take an oath or affirmation to the effect that as President of the Philippines, he will, among others,
"execute its laws."116 In the exercise of such function, the President, if needed, may employ the powers
attached to his office as the Commander-in-Chief of all the armed forces of the country,117 including the
Philippine National Police 118 under the Department of Interior and Local Government.119
This Court rules that the assailed PP 1017 is unconstitutional insofar as it grants President
Arroyo the authority to promulgate "decrees." Legislative power is peculiarly within the province of
the Legislature. Section 1, Article VI categorically states that "[t]he legislative power shall be vested
in the Congress of the Philippines which shall consist of a Senate and a House of
Representatives." To be sure, neither Martial Law nor a state of rebellion nor a state of emergency can
justify President Arroyos exercise of legislative power by issuing decrees.
Can President Arroyo enforce obedience to all decrees and laws through the military?
As this Court stated earlier, President Arroyo has no authority to enact decrees. It follows that these
decrees are void and, therefore, cannot be enforced. With respect to "laws," she cannot call the military to
enforce or implement certain laws, such as customs laws, laws governing family and property relations,
laws on obligations and contracts and the like. She can only order the military, under PP 1017, to enforce
laws pertinent to its duty to suppress lawless violence.
The pertinent provision of PP 1017 states:
x x x and to enforce obedience to all the laws and to all decrees, orders, and regulations promulgated by
me personally or upon my direction; and as provided in Section 17, Article XII of the Constitution
do hereby declare a state of national emergency.
The import of this provision is that President Arroyo, during the state of national emergency under PP
1017, can call the military not only to enforce obedience "to all the laws and to all decrees x x x" but also
to act pursuant to the provision of Section 17, Article XII which reads:
Sec. 17. In times of national emergency, when the public interest so requires, the State may, during the
emergency and under reasonable terms prescribed by it, temporarily take over or direct the operation of
any privately-owned public utility or business affected with public interest.
What could be the reason of President Arroyo in invoking the above provision when she issued PP 1017?
The answer is simple. During the existence of the state of national emergency, PP 1017 purports to grant
the President, without any authority or delegation from Congress, to take over or direct the operation of
any privately-owned public utility or business affected with public interest.
Generally, Congress is the repository of emergency powers. This is evident in the tenor of Section
23 (2), Article VI authorizing it to delegate such powers to the President. Certainly, a body cannot
delegate a power not reposed upon it. However, knowing that during grave emergencies, it may not
be possible or practicable for Congress to meet and exercise its powers, the Framers of our Constitution
deemed it wise to allow Congress to grant emergency powers to the President, subject to certain
conditions, thus:
(1) There must be a war or other emergency.
(2) The delegation must be for a limited period only.
(3) The delegation must be subject to such restrictions as the Congress may prescribe.

Updated Champ Digests

Fel. Jill. Lauren. Mon. Tara. Charms

POLITICAL LAW REVIEW


Atty. Jacinto Jimenez
(4) The emergency powers must be exercised to carry out a national policy declared by Congress.
The point is, under this framework of government, legislation is preserved for Congress all the time, not
excepting periods of crisis no matter how serious. Never in the history of the United States, the basic
features of whose Constitution have been copied in ours, have specific functions of the legislative branch
of enacting laws been surrendered to another department unless we regard as legislating the carrying
out of a legislative policy according to prescribed standards; no, not even when that Republic was fighting
a total war, or when it was engaged in a life-and-death struggle to preserve the Union. The truth is that
under our concept of constitutional government, in times of extreme perils more than in normal
circumstances the various branches, executive, legislative, and judicial, given the ability to act, are called
upon to perform the duties and discharge the responsibilities committed to them respectively."
Following our interpretation of Section 17, Article XII, invoked by President Arroyo in issuing PP 1017, this
Court rules that such Proclamation does not authorize her during the emergency to temporarily take over
or direct the operation of any privately owned public utility or business affected with public interest
without authority from Congress.
Let it be emphasized that while the President alone can declare a state of national emergency, however,
without legislation, he has no power to take over privately-owned public utility or business affected with
public interest. The President cannot decide whether exceptional circumstances exist warranting the take
over of privately-owned public utility or business affected with public interest. Nor can he determine when
such exceptional circumstances have ceased. Likewise, without legislation, the President has no power
to point out the types of businesses affected with public interest that should be taken over. In short, the
President has no absolute authority to exercise all the powers of the State under Section 17, Article VII in
the absence of an emergency powers act passed by Congress.
c. As Applied Challenge
One of the misfortunes of an emergency, particularly, that which pertains to security, is that military
necessity and the guaranteed rights of the individual are often not compatible. Our history reveals that in
the crucible of conflict, many rights are curtailed and trampled upon. Here, the right against
unreasonable search and seizure; the right against warrantless arrest; and the freedom of
speech, of expression, of the press, and of assembly under the Bill of Rights suffered the greatest
blow.
Can this Court adjudge as unconstitutional PP 1017 and G.O. No 5 on the basis of these illegal acts? In
general, does the illegal implementation of a law render it unconstitutional?
Settled is the rule that courts are not at liberty to declare statutes invalid although they may be abused
and misabused135 and may afford an opportunity for abuse in the manner of application.136 The
validity of a statute or ordinance is to be determined from its general purpose and its efficiency to
accomplish the end desired, not from its effects in a particular case.137 PP 1017 is merely an
invocation of the Presidents calling-out power. Its general purpose is to command the AFP to suppress all
forms of lawless violence, invasion or rebellion. It had accomplished the end desired which prompted
President Arroyo to issue PP 1021. But there is nothing in PP 1017 allowing the police, expressly or
impliedly, to conduct illegal arrest, search or violate the citizens constitutional rights.
Now, may this Court adjudge a law or ordinance unconstitutional on the ground that its implementor
committed illegal acts? The answer is no. The criterion by which the validity of the statute or ordinance is
to be measured is the essential basis for the exercise of power, and not a mere incidental result
arising from its exertion.138 This is logical. Just imagine the absurdity of situations when laws maybe
declared unconstitutional just because the officers implementing them have acted arbitrarily. If this were
so, judging from the blunders committed by policemen in the cases passed upon by the Court, majority of
the provisions of the Revised Penal Code would have been declared unconstitutional a long time ago.
Significantly, there is nothing in G.O. No. 5 authorizing the military or police to commit acts beyond what
are necessary and appropriate to suppress and prevent lawless violence, the limitation of their
authority in pursuing the Order. Otherwise, such acts are considered illegal.
SUMMATION

Updated Champ Digests

Fel. Jill. Lauren. Mon. Tara. Charms

POLITICAL LAW REVIEW


Atty. Jacinto Jimenez
In sum, the lifting of PP 1017 through the issuance of PP 1021 a supervening event would have
normally rendered this case moot and academic. However, while PP 1017 was still operative, illegal acts
were committed allegedly in pursuance thereof. Besides, there is no guarantee that PP 1017, or one
similar to it, may not again be issued. Already, there have been media reports on April 30, 2006 that
allegedly PP 1017 would be reimposed "if the May 1 rallies" become "unruly and violent." Consequently,
the transcendental issues raised by the parties should not be "evaded;" they must now be resolved to
prevent future constitutional aberration.
The Court finds and so holds that PP 1017 is constitutional insofar as it constitutes a call by the President
for the AFP to prevent or suppress lawless violence. The proclamation is sustained by Section 18, Article
VII of the Constitution and the relevant jurisprudence discussed earlier. However, PP 1017s extraneous
provisions giving the President express or implied power (1) to issue decrees; (2) to direct the AFP to
enforce obedience to all laws even those not related to lawless violence as well as decrees promulgated
by the President; and (3) to impose standards on media or any form of prior restraint on the press, are
ultra vires and unconstitutional. The Court also rules that under Section 17, Article XII of the
Constitution, the President, in the absence of a legislation, cannot take over privately-owned public utility
and private business affected with public interest.
In the same vein, the Court finds G.O. No. 5 valid. It is an Order issued by the President acting as
Commander-in-Chief addressed to subalterns in the AFP to carry out the provisions of PP 1017.
Significantly, it also provides a valid standard that the military and the police should take only the
"necessary and appropriate actions and measures to suppress and prevent acts of lawless
violence."But the words "acts of terrorism" found in G.O. No. 5 have not been legally defined and made
punishable by Congress and should thus be deemed deleted from the said G.O. While "terrorism" has been
denounced generally in media, no law has been enacted to guide the military, and eventually the courts,
to determine the limits of the AFPs authority in carrying out this portion of G.O. No. 5.
On the basis of the relevant and uncontested facts narrated earlier, it is also pristine clear that (1) the
warrantless arrest of petitioners Randolf S. David and Ronald Llamas; (2) the dispersal of the rallies and
warrantless arrest of the KMU and NAFLU-KMU members; (3) the imposition of standards on media or any
prior restraint on the press; and (4) the warrantless search of the Tribune offices and the whimsical
seizures of some articles for publication and other materials, are not authorized by the Constitution, the
law and jurisprudence. Not even by the valid provisions of PP 1017 and G.O. No. 5.
Other than this declaration of invalidity, this Court cannot impose any civil, criminal or administrative
sanctions on the individual police officers concerned. They have not been individually identified and given
their day in court. The civil complaints or causes of action and/or relevant criminal Informations have not
been presented before this Court. Elementary due process bars this Court from making any specific
pronouncement of civil, criminal or administrative liabilities.
It is well to remember that military power is a means to an end and substantive civil rights are
ends in themselves. How to give the military the power it needs to protect the Republic without
unnecessarily trampling individual rights is one of the eternal balancing tasks of a democratic
state.During emergency, governmental action may vary in breadth and intensity from normal times, yet
they should not be arbitrary as to unduly restrain our peoples liberty.
Perhaps, the vital lesson that we must learn from the theorists who studied the various competing political
philosophies is that, it is possible to grant government the authority to cope with crises without
surrendering the two vital principles of constitutionalism: the maintenance of legal limits to arbitrary
power, and political responsibility of the government to the governed.158
WHEREFORE, the Petitions are partly granted. The Court rules that PP 1017 is CONSTITUTIONAL
insofar as it constitutes a call by President Gloria Macapagal-Arroyo on the AFP to prevent or suppress
lawless violence. However, the provisions of PP 1017 commanding the AFP to enforce laws not related to
lawless violence, as well as decrees promulgated by the President, are declared UNCONSTITUTIONAL.
In addition, the provision in PP 1017 declaring national emergency under Section 17, Article VII of the
Constitution is CONSTITUTIONAL, but such declaration does not authorize the President to take over
privately-owned public utility or business affected with public interest without prior legislation.
G.O. No. 5 is CONSTITUTIONAL since it provides a standard by which the AFP and the PNP should
implement PP 1017, i.e. whatever is "necessary and appropriate actions and measures to suppress
and prevent acts of lawless violence." Considering that "acts of terrorism" have not yet been defined
Updated Champ Digests

Fel. Jill. Lauren. Mon. Tara. Charms

POLITICAL LAW REVIEW


Atty. Jacinto Jimenez
and made punishable by the Legislature, such portion of G.O. No. 5 is declared UNCONSTITUTIONAL.
The warrantless arrest of Randolf S. David and Ronald Llamas; the dispersal and warrantless arrest of the
KMU and NAFLU-KMU members during their rallies, in the absence of proof that these petitioners were
committing acts constituting lawless violence, invasion or rebellion and violating BP 880; the imposition of
standards on media or any form of prior restraint on the press, as well as the warrantless search of the
Tribune offices and whimsical seizure of its articles for publication and other materials, are declared
UNCONSTITUTIONAL.

SECTION 19

DRILON VS. CA
reopening of a case after pardon

1970s: Mayor Rodolfo Ganzon charged with double murder in the military courts. Sentenced to life
imprisonment with hard labor. He later joined KBL (Marcos party) where he was designated as
campaign manager. (Because of this, his sentence was reduced to 6 years)
After Edsa I, then justice secretary Ordonez wanted to conduct preliminary investigation against
Ganzon for the murders.
Ganzon claimed that he had been extended an absolute pardon by President Marcos, and having been
previously convicted, he cannot be tried anew.
The RTC denied the MTD, but the CA granted the MTD.
Drilon, as the new justice secretary claims that there exists no evidence to prove that Ganzon was
pardoned.

ISSUE: Can Ganzon still be prosecuted?


SC: NO MORE.
The record shows that Ganzon had already been arraigned by the military court, pleaded not guilty, but
convicted and sentenced. He served time until 1978, when he was placed under HOUSE ARREST by
President Marcos.
Since Ganzon served already 6 years and released, the court cannot tolerate any further reinvestigation.
The 6-year service of sentence and subsequent release are significant since if President Marcos ordered
Ganzons release after 6 years of imprisonment, Marcos unavoidably commuted Ganzons life
imprisonment to 6 years.
If Ganzons sentence had already been commuted, he has therefore served his sentence fully, and if he
has served his sentence fully, he can no longer be reinvestigated. Under the Constitution, the pardoning
power of the President is final and unappealable. So is commutation of sentence, in which the President
reduces a sentence. It extinguishes criminal partially, and has the effect of changing the penalty to a
lesser one.
The commutation of sentence need not be in a specific form. It is sufficient that Ganzon was voluntarily
released in 1978 with no terms or conditions, except that he should remain in house arrest. This house
arrest cannot be considered as a continuation of his sentence, because in no way is arrest a penalty, but
rather a mere means of taking a person into custody. Also, the exact terms of the house arrest are not
known.
Hence, irrespective of the pardon, Ganzon has served his sentence and cannot be reinvestigated for the
same offense, much more undergo further imprisonment to complete his sentence.
The fact that Ganzon might have gotten off too lightly is immaterial. We cannot ignore the legal effects of
Marcos acts.
CRISTOBAL VS. LABRADOR.
Purpose of executive clemency

Teofilo Santos was found guilty of estafa, sentenced to 6 mos. However he continued to be a
registered elector, seated as the municipal president in Malabon

Updated Champ Digests

Fel. Jill. Lauren. Mon. Tara. Charms

POLITICAL LAW REVIEW


Atty. Jacinto Jimenez

When the Election Code was passed, it disqualified from voting, persons who are guilty of any crime
against property.
Santos thus applied for absolute pardon from the President. This was granted and he was restored to
full civil and political rights, EXCEPT with respect to the right to hold public office. In short, he was
only eligible to clerical positions, which do not involve money or property.
Cristobal however filed a petition for exclusion of Santos from the list of voters, contending the Santos
is still disqualified. Crisotobal argues that the pardon granted by the President did not restore him to
full enjoyment of his political rights because:

A) the pardoning power of the President does not apply to legislative prohibitions

B) the pardoning power amounts to unlawful exercise by the President of a


legislative function

C) Santos has served his sentence and there is nothing to pardon

ISSUE: What is the nature of Santos pardon by the President?


SC:
In the present case, while the pardon extended to Santos is conditional in the sense that he can only be
appointed to clerical or manual positions, IT IS STILL ABSOULTE INSOFAR AS IT RESTORES SANTOS TO
FULL AND CIVIL POLITICAL RIGHTS.
There are 2 limitations on the Exercise of Pardoning Power
1) power exercised after conviction
2)power does not extend to impeachment cases
The Pardoning Power cannot be restricted or controlled by legislative action. It can only be subject to
constitutional limitations.
An absolute pardon erases not only the crime committed but also removes all disabilities resulting from
conviction. Thus it extends to accessory and resultant disabilities. When it is granted after conviction it
removes all that is left of the consequences of the conviction.

TORRES VS. GONZALEZ


violation of conditional pardon

Torres convicted of Estafa. He was granted conditional pardon by the President. Condition was that he
should not violate any of the penal laws, if he does, he will be proceeded against in the manner
prescribed by law.
In 1986, the Board of Pardons and Parole recommended the conditional pardon because: 1) he was
charged for 20 counts of estafa, 2) he was convicted of sedition but pending appeal to the CA, 3)
according to the NBI report, he haad a long list of charged for various crimes.
The President thus cancelled the conditional pardon. The minister of justice issued the order of arrest
and recommitment.
Torres now assails the validity of this Order of Arrest. He claims that he did not violate the conditions
of his parole since he has not been convicted by final judgment of the 20 counts of estafa, nor of the
crime of sedition. He also claims that he was not given the opportunity to be heard.

ISSUE: Whether conviction of a crime by final judgment of a court is necessary before the petitioner can
validly be re-arrested and re-committed for violation of the terms of his conditional pardon?
SC: NO.
The grant of pardon and the determination of the terms and conditions of the pardon are purely executive
acts, and are not subject to judicial review. The determination of the breach of conditional pardon may
either be:
1) purely executive act not subject to judicial review. Here, no judicial pronouncement of guilt is
necessary. Conviction by final judgment is not necessary.
2) a judicial act consists of trial and conviction of violation of conditional pardon.
Here, where the President opts to proceed under Sec 64 of the Revised Administrative Code, no judicial
pronouncement of guilt is necessary, much less a conviction by final judgment of a court, in order that a
convict may be recommended for the violation of his conditional pardon.

Updated Champ Digests

Fel. Jill. Lauren. Mon. Tara. Charms

POLITICAL LAW REVIEW


Atty. Jacinto Jimenez
What is involved in this case, is not the prosecution of the parolee for a subsequent offense. What is
involved here is rather the ascertainment of whether the convict has breached the conditions of his parole,
that he would not again violate any of the penal laws, for purposes of reimposition upon him of the
remitted portion of his original sentence. The consequences that we here deal with are the consequence of
an ascertained breach of the conditions of pardon.
In sum, when proceeding against a convict who has been conditionally pardoned but have breached the
conditions, the President has two options:
1) proceed against him under the Revised Admin Code
2) proceed against him under Art 159 of the RPC, which imposes the penalty of prision correctional for
having violated a conditional pardon.
Here the President chose #1. That choice is an exercise of the Presidents executive prerogative and NOT
SUBJECT TO JUDICIAL SCRUTINY.
MONSATO VS. FACTORAN
pardon: nature and legal effects, reinstatement to former rights.

Monsato was convicted of estafa thru falsification of public documents. President Marcos extended
absolute pardon.
By reason of such pardon, she wrote the Calbayog City Treasurer, requesting that she be restored to
her former post as assistant city treasurer.
The matter was referred to the Minister of Finance. It was ruled that she may be reinstated without
need for a new appointment but not earlier than the date she was extended the absolute pardon.
Monsato however insisted that full pardon had wiped out the crime, such that her service in the
government has never been interrupted and therefore her reinstatement should correspond to the
date of her preventive suspension. Monsata further argues that the pardon was granted to her before
the final verdict of guilt when the conviction was pending appeal. Monsato thus claims that it was
equivalent to acquittal because there was no offense to speak of.
The matter was referred to the President/ Executive Secretary Factoran, who ruled that Monsata was
not entitled to automatic reinstatement.

ISSUE: Does Monsatos pardon have the effect of acquittal?


SC: NO. FACTORAN WAS CORRECT.
PARDON IMPLIES GUILT. While it relieves the party pardoned from all the punitive consequences of his
criminal act, it relieves him from nothing more. A pardon looks into the future, it does not impose upon
the government any obligation to make reparation for what has been suffered. Since the offense has been
established by judicial proceedings, that which has been done or suffered is presumed to have been
rightfully done and justly suffered.
Pardon is an act of grace proceeding from the power entrusted with the execution of laws, which exempts
the individual from punishment for a crime he has committed. The very essence of pardon is forgiveness
or remission of guilt. It does not erase the fat of commission of crime and the conviction thereof. IT DOES
NOT WASH OUT THE MORAL STAIN. IT INVOLVES FORGIVENESS AND NOT FORGETFULLNESS. (naks!)
Thus, even if Monsato was pardoned, she CANNOT BE ENTITLED TO BACKPAY FOR LOST EARNINGS.
*Here, we do NOT subscribe to the fictitious belief that pardon blots out the guilt of an individual and that
once he is absolved, he should be treated as if he were innocent.
There cannot also be automatic reinstatement because pardon does not ipso facto restore a convicted
felon to public office which was forfeited by reason of the conviction. Such pardon merely restores his
eligibility for appointment to that office.
A pardon, even if full and plenary, cannot preclude the appointing power from refusing appointment to
anyone deemed to be of bad character. The absolute disqualification or ineligibility from public office forms
part of the punishment imposed by the RPC. The pardon granted results only in removing her
disqualification and cannot go beyond that. She must therefore RE-APPLY AND UNDERGO THE USUAL
PROCEDURE FOR A NEW APPOINTMENT.

LLAMAS VS. ORBOS


constitutional limits of executive clemency

Updated Champ Digests

Fel. Jill. Lauren. Mon. Tara. Charms

POLITICAL LAW REVIEW


Atty. Jacinto Jimenez

Llamas was Vice Governor of Tarlac. He assumed the position as Governor because Governor Ocampo
was suspended from office.
Governor Ocampo was previously charged with graft and corruption before the ombudsman in
connection with the Loan Agreement made with Lingkod Tarlac Foundation, which was allegedly not
approved by the provincial board.
Governor Ocampo was also charged before the Department of Local Government.
Governor Ocampo was found guilty and was suspended for 90 days. He appealed to the Office of the
President. But Executive Secretary Orbos denied it.
Now, when Llamas assumed as governor, Ocampo still issued an administrative order signifying his
intention to continue his office at his residence.
Orbos reduced the 90-day suspension to time already served. In short, Ocampo was granted
executive clemency because of his eagerness to deliver livelihood projects.
So Ocampo re-assumed governorship.
Llamas assails the resolution of Executive Secretary Orbos, claiming it was GADALEJ. Llamas claims
that executive clemency can only be granted in criminal case, not administrative cases

ISSUE: Can the President grant executive clemency in administrative cases? Does the word conviction
by final judgment refer to criminal or administrative case?
SC:
UBI LEX NON DISTINGUIT, NEC NOS DISTINGUIRE DEBEMOS. When the law does not distinguish neither
shall we. The Constitution does not distinguish between which cases executive clemency may be exercised
by the President. (except only in cases of impeachment). If administrative cases were not included, then
why would the constitution state that the only exception would be impeachment cases (which is not a
criminal case).
If those adjudged guilty criminally may be pardon there is no reason why the same benefit may not be
extended to those adjudged guilty administratively. The president can grant executive clemency in
administrative cases, which are clearly less serious than criminal offenses.
Also, by virtue of the POWER OF SUPERVISION AND CONTROL, the President may reverse or modify a
ruling issued by a subordinate against an erring public official.
Note also that upon acceptance of pardon, the grantee is deemed to have waived any appeal which he
may have filed.

SECTION 21
GONZALES VS. HECHANOVA

Executive Secretary authorized the importation of foreign rice to be purchased from private sources.
(The was a Procurement Committee created.)
Gonzales, a rice planter, filed a petition question the validity of the attempt to import foreign rice. He
contends it is against RA 3452 which prohibits importation of rice and corn by the Rice and Corn
Admin or any other govt agency.
Hechanova countered that the importation is authorized by the President for military stock pile
purposes. (that the president is duty-bound to prepare for the challenge of threats of war or
emergency without waiting for special authority)
Hechanova further contends that there is no prohibition on importation made by the Government
itself.
Hechanova also argued that the Government has already entered into 2 contracts with Vietnam and
Burma. That these contracts constitute valid executive agreements under international law; that such
agreements became binding and effective upon signing thereof by the representatives of both parties.
o
It is argued that when there is a conflict between a treaty and a statute (the statute
prohibiting importation), then the conflict must be resolved in favor of the one which is latest
in point of time (in this case, the treaty).

ISSUE: What is the nature of the government contracts with Vietnam and Burma? Which should prevail,
the contracts or RA 3452
SC:

Updated Champ Digests

Fel. Jill. Lauren. Mon. Tara. Charms

POLITICAL LAW REVIEW


Atty. Jacinto Jimenez
The parties to said contracts do not appear to have regarded the same as executive agreements. Even
assuming that said contracts are executive agreements, they are null and void, because they are
inconsistent with RA 3452.
Although the President may enter into executive agreements without previous legislative authority, he
may not, by executive agreement, enter into a transaction which is prohibited by statutes enacted prior
thereto. Under the Constitution, the main function of the Executive is to enforce the laws enacted by
Congress. He may not defeat legislative enactments by indirectly repealing the same through an executive
agreement providing for the performance of the very act prohibited by said laws.
**Also, the Supreme Court has jurisdiction over the case. The Constitution authorizes the nullification of a
treaty not only when it conflicts with the fundamental law, but also when it runs counter to the act of
Congress.
The E.S. has no power to authorize the importation.
USAFFE VS. TREASURER

The petitioners assail the validity of the Romulo Snyder Agreement of 1950, signed in Washington.
The $35M remained unexpended and in the possession of the AFP. President Quirino thru Central Bank
Governor proposed to US officials the retention of the $35M as a loan.
Under the Agreement, the Philippine govt undertook to return to the US in 10 annual installments a
total of $35M advanced by the US, but was not spent by the Philippines National Defense forces.
The Usaffe Veterans claim that the agreement is illegal because they are funds already appropriated
by American Congress for the Philippine Army. They also contend that US Secretary of Treasury
Synder and Philippine Foreign Secretary Carlos P Romulo both have no authority in entering into the
agreement.
They contend that the agreement lacks the ratification by the Philippine Senate to make it binding on
Philippine Government.
On the other hand, the government contends that the agreement is not a treaty, but an executive
agreement, and therefore it does not require ratification of Senate.

ISSUE: What is the nature of the Agreement?


SC:
Executive agreements are of two classes:
1) agreement made purely as executive acts affecting externatl relations and independent of or needs no
legislative authorization. (Presidential Agreements)
2) agreement entered into in pursuance of acts of Congress, (Congressional-Executive Agreements)
The Agreement may fall under any of these 2 classes. Why?
1) Because Congress granted authority to the President to obtain loans and incur indebtedness with the
Government of the US.
2) Even assuming that there was no legislative authorization, the Agreement may still be entered into
purely as executive acts. (which usually relates to money agreements for settlement of pecuniary claims
of citizens)
3) Senate Resolution # 15 admitted the validity and binding force of the agreement.
4) The act of Congress appropriating funds for the yearly installments (under the Agreement) constitute a
ratification thereof.
Hence, the validity of the Agreement is beyond the Courts reach.

Updated Champ Digests

Fel. Jill. Lauren. Mon. Tara. Charms

You might also like